Đến nội dung

Hình ảnh

$\left ( p-1 \right )^{n}+1\vdots n^{p-1}$

- - - - -

  • Please log in to reply
Chủ đề này có 4 trả lời

#1
NTMFlashNo1

NTMFlashNo1

    Sĩ quan

  • Thành viên
  • 344 Bài viết

Tìm các cặp (n;p) nguyên dương ; p là số nguyên tố thỏa mãn:

                                                                      

                                                        $\left ( p-1 \right )^{n}+1\vdots n^{p-1}$


$\boxed{\text{Nguyễn Trực-TT-Kim Bài secondary school}}$


#2
yeutoan2001

yeutoan2001

    Thượng sĩ

  • Thành viên
  • 231 Bài viết

Đề thiếu $n\leq 2p$ thì phải



#3
k4x

k4x

    Binh nhì

  • Thành viên mới
  • 13 Bài viết

Đề thiếu $n\leq 2p$ thì phải

lời giải của mình (mình nghĩ cũng chặn điều kiện là $n \leq 2p$)

trường hợp đơn giản đầu tiên đó là $p=2$ => $n=2$

khi $p$ lẻ ta xử lý như sau

để thuận tiện cho việc theo dõi thì ta đặt $p-1=x$ , dễ thấy $n$ lẻ 

ta có: $x \equiv -1$ (mod x+1) cho nên $\sum_{i=1}^{n-1}(-1)^{i}x^{n-i}$ $\equiv$ $n$ (mod x+1) $(1)$

$x^n+1=(x+1).\sum_{i=1}^{n-1}(-1)^{i}x^{n-i}$ cho nên từ $(1)$ ta có 2 trường hợp sau:

Th1: $n \equiv 0$ (mod x+1) nên $n=p$ ($n$ khác $2p$ vì $n$ lẻ)

       $(p-1)^n+1 \vdots n^{p-1}$ => $v_{p}(VT) \geq v_{p}(VP)$ => $2 \geq p-1$ => $p=3$ 

       vậy $(p,n)=(3,3)$

Th2: $n$ không chia hết cho $p$

khi đó vì $p=x+1$ là số nguyên tố cho nên $gcd(p;\sum_{i=1}^{n-1}(-1)^{i}x^{n-i})=1$ $(2)$

đặt $(p-1)^n+1=n^{p-1}.k$ từ $(2)$ suy ra $gcd(n^{p-1},k)=1$ nên ta tiếp tục có 2 trường hợp nhỏ nữa

    Th3:$p=x+1=n^{p-1}$, vì $p$ là số nguyên tố nên hoặc $n=1$ => pt có nghiệm $(p,1)$

                                                                                         hoặc $p=2$ (loại)

    Th4:$\sum_{i=1}^{n-1}(-1)^{i}x^{n-i}=n^{p-1} \equiv n$ (mod p) $(3)$

            vì $n$ không chia hết cho $p$ nên theo định lý $Fermat$ $n^{p-1} \equiv 1$ (mod p) $(4)$

            Từ $(3),(4)$ suy ra $n-1 \vdots p$ => $n=p+1$ => $n$ chẵn (vô lý)

Vậy phương trình có các nghiệm sau $(p;n)$ $\in$ ${(2;2),(3;3),(p,1)}$ 


Bài viết đã được chỉnh sửa nội dung bởi k4x: 30-11-2016 - 12:49


#4
NTMFlashNo1

NTMFlashNo1

    Sĩ quan

  • Thành viên
  • 344 Bài viết

lời giải của mình (mình nghĩ cũng chặn điều kiện là $n \leq 2p$)

trường hợp đơn giản đầu tiên đó là $p=2$ => $n=2$

khi $p$ lẻ ta xử lý như sau

để thuận tiện cho việc theo dõi thì ta đặt $p-1=x$ , dễ thấy $n$ lẻ 

ta có: $x \equiv -1$ (mod x+1) cho nên $\sum_{i=1}^{n-1}(-1)^{i}x^{n-i}$ $\equiv$ $n$ (mod x+1) $(1)$

$x^n+1=(x+1).\sum_{i=1}^{n-1}(-1)^{i}x^{n-i}$ cho nên từ $(1)$ ta có 2 trường hợp sau:

Th1: $n \equiv 0$ (mod x+1) nên $n=p$ ($n$ khác $2p$ vì $n$ lẻ)

       $(p-1)^n+1 \vdots n^{p-1}$ => $v_{p}(VT) \geq v_{p}(VP)$ => $2 \geq p-1$ => $p=3$ 

       vậy $(p,n)=(3,3)$

Th2: $n$ không chia hết cho $p$

khi đó vì $p=x+1$ là số nguyên tố cho nên $gcd(p;\sum_{i=1}^{n-1}(-1)^{i}x^{n-i})=1$ $(2)$

đặt $(p-1)^n+1=n^{p-1}.k$ từ $(2)$ suy ra $gcd(n^{p-1},k)=1$ nên ta tiếp tục có 2 trường hợp nhỏ nữa

    Th3:$p=x+1=n^{p-1}$, vì $p$ là số nguyên tố nên hoặc $n=1$ => pt có nghiệm $(p,1)$

                                                                                         hoặc $p=2$ (loại)

    Th4:$\sum_{i=1}^{n-1}(-1)^{i}x^{n-i}=n^{p-1} \equiv n$ (mod p) $(3)$

            vì $n$ không chia hết cho $p$ nên theo định lý $Fermat$ $n^{p-1} \equiv 1$ (mod p) $(4)$

            Từ $(3),(4)$ suy ra $n-1 \vdots p$ => $n=p+1$ => $n$ chẵn (vô lý)

Vậy phương trình có các nghiệm sau $(p;n)$ $\in$ ${(2;2),(3;3),(p,1)}$ 

Lời giải của bạn HAY

BẠn thử làm bài này xem

http://diendantoanho...ft-p-1-right-k/

Mình dùng Langreder nhưng chưa ra


$\boxed{\text{Nguyễn Trực-TT-Kim Bài secondary school}}$


#5
I Love MC

I Love MC

    Đại úy

  • Thành viên nổi bật 2016
  • 1861 Bài viết

Tìm các cặp (n;p) nguyên dương ; p là số nguyên tố thỏa mãn:

                                                                      

                                                        $\left ( p-1 \right )^{n}+1\vdots n^{p-1}$

$n=1$ thì $p$ là số nguyên tố bất kì thỏa yêu cầu 
$n \ge 2$ thì xét 
TH1 : $p=2 \Rightarrow n=2$
TH2 : $p$ lẻ ,gọi $q$ là ước nguyên tố nhỏ nhất của $n$. 
Từ giả thiết ta có $(p-q)^{2n} \equiv 1 \pmod{q},(p-1,q)=1$ 
Đặt $ord_q(p-1)=t \Rightarrow t|2n$ . Sử dụng định lý Fermat có ngay $t|q-1$ 
Vậy ta thu được $t|2n,t|q-1$ > Nếu $(t,n)=r>1$ thì $r|n$ và $t \ge r$ . Mà $t|q-1$ nên $t<q$ suy ra $r<q$ . Lại do $r,q$ đều là ước nguyên tố của $n$ nên mâu thuẫn điều kiện nhỏ nhất của $q$ . 
Vậy $(t,n)=1 \Rightarrow t|2$ . Ta thu được $(p-1)^2 \equiv 1 \pmod{q}$ hay $q|p(p-2)$ . 
+) Nếu $q|p-2 \Rightarrow (p-1)^n+1 \equiv 2 \pmod{q} \Rightarrow q=2$ . Ta có $(p-1)^n \equiv -1 \pmod{2} \Rightarrow p=2$ mâu thuẫn vì $p$ lẻ 
+) $q|p$ . Khi $n$ chẵn thì dễ thấy vô lí nên $n$ lẻ  
Áp dụng LTE : $v_q((p-1)^n+1)=v_q(n)+v_q(p) \ge v_q(n)(p-1)$ 
Đặt $p=q^ab,a,b \in \mathbb{N^*}$ . Ta có kết quả sau đây 
$q^ab \ge a+2$ với $q \ge 3$ do $q|p$. Dấu bằng xảy ra $\Leftrightarrow a=b=1,q=3 \Rightarrow p \ge v_q(p)+2$ 
$\Rightarrow p-2 \ge v_q(p) \ge v_q(n)(p-2)$ 
Vậy $q=p=3$ và $v_3(n)=1$ 
Đặt $n=3k$ trong đó $(k,3)=1$ 
Từ đề bài ta sẽ có $9k^2|8^k+1$  . Vì $8^k+1 \vdots 9$ do $k$ lẻ nên việc tìm $k$ để thỏa mãn cái này là tương đương với tìm $k$ để : 
$k^2|8^k+1$ 
Với $k=1 \Rightarrow n=3$ . Với $k \ge 2$ tương tự như cách chứng minh trên ta thấy $r$ là ước nguyên tố nhỏ của $k$ và $s=ord_r(8)$ 
Suy ra $s|2 \Rightarrow s=2$ . Khi đó $r|8^2-1 \Rightarrow r|7$ vô lí vì $8^k+1 \equiv 2 \pmod{7}$ 
Vậy $(n,p)=(1,p),(2,2),(3,3)$






0 người đang xem chủ đề

0 thành viên, 0 khách, 0 thành viên ẩn danh